Partielle Integration Wo ist mein Fehler

Aufrufe: 493     Aktiv: 08.02.2021 um 13:56

0
Hallo, ich möchte die Varianz der Funktion \( \int_0^∞ \lambda * e^{-\lambda x}\) ausrechnen,
doch habe ein Fehler bei der Partielle Integration. Kann mir jemand weiterhelfen und auch sagen wo der Fehler ist ? 
Danke für Antworten 

das Ergebnis soll sein: \( 1/ \lambda ^2 \)

Diese Frage melden
gefragt

Punkte: 24

 
Kommentar schreiben
1 Antwort
2
Dein Fehler liegt darin, dass du die Varianz nicht nur mit \(\mathbb{E}(X^2)\) bestimmst. Es gilt:
\(Var(X)=\mathbb{V}(X)=\mathbb{E}(X^2)-(\mathbb{E}(X))^2\neq \mathbb{E}(X^2)\).
Der Erwartungswert der Exponentialverteilung ist \(\mathbb{E}(X)=\dfrac{1}{\lambda}\).
Du erhälst für deine Rechnung durch zweimalige partielle Integration:
\(\displaystyle{\int_0^{\infty} x^2\cdot \lambda^{-\lambda x} dx =\underset{=0}{\underbrace{\left[x^2\cdot \left(-e^{-\lambda x}\right)\right]_0^{\infty}}} -\int_0^{\infty} 2x\cdot \left(-e^{-\lambda x}\right) dx=\int_0^{\infty} 2x\cdot e^{-\lambda x} dx=\underset{=0}{\underbrace{\left[2x\cdot \left(-\dfrac{1}{\lambda} e^{-\lambda x}\right)\right]_0^{\infty}}} -\int_0^{\infty} 2\cdot \left(-\dfrac{1}{\lambda} e^{-\lambda x}\right) dx=\int_0^{\infty} \dfrac{2}{\lambda} e^{-\lambda x} dx =\ldots =\dfrac{2}{\lambda^2}}\)

Ich würde dir empfehlen auf die Notation zu achten, also berechne die Wert in den eckigen Klammern separat und vergiss das \(dx\) am Ende des Integrals nicht ;)

Wenn du nun noch den Erwartungswert \(\mathbb{E}(X)=\dfrac{1}{\lambda}\) einer exponentialverteilten Zufallsvariable \(X\) mit hinzuziehst, erhälst du ein dein gewünschtes Ergebnis durch:
\(Var(X)=\mathbb{E}(X^2)-(\mathbb{E}(X))^2=\dfrac{2}{\lambda^2}- \left(\dfrac{1}{\lambda}\right)^2=\boxed{\dfrac{1}{\lambda^2}}\)


Hoffe das hilft weiter.
Diese Antwort melden
geantwortet

Punkte: 8.93K

 

Vielen dank für deine ausführliche Antwort. Ich hatte schon den Erwartungswert ausgerechnet und wollte deswegen nur noch EX^2 ausrechnen. Ich habe aber noch eine Frage: Wie erkennt man ob \( x^2 (-e^{-\lambda x})\) = 0 ist ? Durch einsetzen von 0 für x ist klar, dass es 0 ergibt, aber wie sieht es mit ∞ aus ? Einfach große Zahlen eingeben ?   ─   danny96 08.02.2021 um 12:49

Du setzt ein \(k\in \mathbb{R}\) ein und betrachtest den Grenzwert gegen unendlich, also \(\underset{k\longrightarrow \infty}{\lim} -\dfrac{k^2}{e^{kx}}\). Da ein exponentieller Ausdruck immer schneller wächst als ein polynomineller, ist der Grenzwert Null ;)   ─   maqu 08.02.2021 um 13:32

vielen vielen Dank   ─   danny96 08.02.2021 um 13:50

immer gern ;)   ─   maqu 08.02.2021 um 13:56

Kommentar schreiben